RNSG 2371 Prep U Questions Exam 4 Tissue Integrity, Sensory Perception, Cognition

Pataasin ang iyong marka sa homework at exams ngayon gamit ang Quizwiz!

A client complains of vertigo. The nurse anticipates that the client may have a problem with which portion of the ear? a) Inner ear b) Tympanic membrane c) External ear d) Middle ear

A

A client states, "My children still need me. Why did I get cancer? I am only 30." This client is exhibiting which stage according to Kübler-Ross? a) Bargaining b) Anger c) Acceptance d) Denial

B

A client with cancer is receiving chemotherapy. The nurse should assess which of the following diagnostic values while the client is receiving chemotherapy? a) Pancreatic enzymes b) Bone marrow cells c) Liver tissues d) Heart tissues

B

The ear canal of an infant or young child: a) slants downward. b) is horizontal. c) slants backward. d) slants upward.

D

The client's expected outcome is "The client will maintain skin integrity by discharge." Which of the following measures is best in evaluating the outcome? a) Pressure-relieving mattress on the bed. b) Condition of the skin over bony prominences. c) Percent intake of a diet high in protein. d) The client's ability to reposition self in bed.

B

A 90-year-old woman has a staphylococcus infection in her decubitus ulcer. Staphylococcus is the: a) Host b) Environment c) Agent d) Disease

C

Which of the following is also known as "jock itch"? a) Tinea pedis b) Tinea cruris c) Tinea corporis d) Tinea unguium

C

When assessing a client with diabetes for diabetic nephropathy, the nurse should determine if the client has: a) Asymptomatic proteinuria. b) Increasing glycosuria. c) Polyuria. d) Ketonuria.

A (Asymptomatic proteinuria is an initial sign of diabetic nephropathy.)

Which layer of the skin is made of primarily adipose tissue? a) Dermis b) Hypodermis c) Epidermis d) Muscle

B

A client claims to have a "special mission from God". The nurse incorporates this religious delusion of grandeur into the client's plan of care based on the understanding that the primary purpose of such a delusion is to provide which of the following? a) Sexual outlet. b) Comfort. c) Self-esteem. d) Safety.

C

Which of the following is an accurate statement regarding refractive surgery? a) Refractive surgery may be performed on all patients, even if they have underlying health conditions. b) Refractive surgery will alter the normal aging of the eye. c) Refractive surgery is an elective, cosmetic surgery performed to reshape the cornea. d) Refractive surgery may be performed on patients with an abnormal corneal structure as long as they have a stable refractive error.

C

Which of the following is an indicator of neurovascular compromise?

Capillary refill >3 seconds

Which of the following is an early indication that a client has developed hypocalcemia? a) Ventricular dysrhythmias. b) Depressed reflexes. c) Memory changes. d) Tingling in the fingers.

D

A client is being treated for acute low back pain. The nurse should report which of these clinical manifestations to the physician immediately? a) Pain in the lower back when the leg is lifted. b) Pain in the lower back that radiates to the hip. c) Diffuse, aching sensation in the L4 to L5 area. d) New onset of footdrop.

D

When shaving a male patient's face, pull the skin taut and use short, upward strokes.

False

Extremity restraints are required to be removed at least once per shift to assess the patient and evaluate the need continued restraint.

False (usually at least every hour.)

Upon assessment of a patient's wound, the nurse notes the formation of granulation tissue. The tissue easily bleeds when the nurse performs wound care. What is the phase of wound healing characterized by the nurse's assessment?

proliferation phase

A 28-year-old female patient presents to the emergency department (ED) stating severe restlessness and anxiety. Upon assessment, the patient's heart rate is 118 bpm and regular, the patient's pupils are dilated, and the patient appears excitable. Which action should the nurse take next? a) Question the patient about alcohol and illicit drug use. b) Instruct the patient to hold her breath and bear down. c) Prepare to administer a calcium channel blocker. d) Place the patient on supplemental oxygen.

A

Which of the following uses the body's own digestive enzymes to break down necrotic tissues? a) Autolytic debridement b) Wet to dry dressings c) Enzymatic debridement d) Wet dressings

A

Which refers to the decrease in lens flexibility that occurs with age, resulting in the near point of focus getting farther away? a) Presbyopia b) Glaucoma c) Cataract d) Presbycusis

A Presbyopia usually begins in the fifth decade of life, when reading glasses are required to magnify objects. Presbycusis refers to age-related hearing loss. Cataract is the development of opacity of the lens of the eye. Glaucoma is a disease characterized by increased intraocular pressure.

Which of the following describes vertigo? Select all that apply. a) Objects are moving around him or her b) Syncope c) Misperception of motion d) Fainting e) Spinning sensation

A, C, E

A client with Raynaud's phenomenon is prescribed diltiazem. An expected outcome is: a) Conversion to normal sinus rhythm. b) Reduced episodes of finger numbness. c) Decreased heart rate. d) Increased SpO2.

B

Which of the following infecting agents is the cause of scabies? a) Bacteria b) Itch mite c) Parasitic fungi d) Reactivated virus

B

A client has a herniated disk in the region of the third and fourth lumbar vertebrae. Which nursing assessment finding most supports this diagnosis? a) Hypoactive bowel sounds b) Weakness and atrophy of the arm muscles c) Severe lower back pain d) Sensory deficits in one arm

C

A client has been perceiving her roommate's stuffed animal as her own dog at home. The nurse determines that this misperception of reality (illusion) is improving when the client makes which of the following statements? a) "I guess Jan needs a dog as much as I do." b) "I wish Jan hadn't had my dog stuffed." c) "Jan's stuffed dog looks somewhat like my dog, Trixie." d) "Jan's dog and my dog could be twins."

C

A patient in a long-term care facility becomes confused and disoriented at night and is incontinent during these periods of confusion due to the inability to find the commode. During the day, the patient does not experience confusion and is continent. What type of incontinence is this patient experiencing during the nighttime hours? a) Transient incontinence b) Reflex incontinence c) Functional incontinence d) Stress incontinence

C Functional incontinence is urine loss caused by the inability to reach the toilet because of environmental barriers, physical limitations, or loss of memory or disorientation. Stress incontinence occurs when there is an involuntary loss of urine related to an increase in intra-abdominal pressure. Reflex incontinence is an emptying of the bladder without the sensation to void. Transient incontinence appears suddenly and lasts for 6 months or less.

It is customary for the registered nurse to perform the initial postoperative dressing change. a) True b) False

B

Dehiscence is the softening of tissue due to excessive moisture.

False

Which of the following is indicative of a right hemisphere stroke? a) Aphasia b) Altered intellectual ability c) Spatial-perceptual deficits d) Slow, cautious behavior

C

Which disturbance results in loss of half of the visual field? a) Diplopia b) Nystagmus c) Anisocoria d) Homonymous hemianopsia

D

The treatment of choice for a spinal cord-injured patient with impaired bladder emptying would include which of the following? a) No catheterization is necessary b) Indwelling catheterization c) Intermittent self-catheterization d) Condom catheterization

C

A nurse assessing the wound healing of a patient, documents that the wound formed a clean, straight line with little loss of tissue. This wound healed by: a) Tertiary intention b) None of the above c) Secondary intention d) Primary intention

D

A client is recovering from a craniotomy with tumor debulking. Which comment by the client indicates to the nurse a correct understanding of what the surgery entailed? a) "I guess the doctor could not remove the entire tumor." b) "Thank goodness the tumor is contained and curable." c) "I will be glad to finally be done with treatments for this thing." d) "I am so glad the doctor was able to remove the entire tumor."

A

A client preparing to undergo a lumbar puncture states he doesn't think he will be able to get comfortable with his knees drawn up to his abdomen and his chin touching his chest. He asks if he can lie on his left side. Which statement is the best response by the nurse? a) "Although the required position may not be comfortable, it will make the procedure safer and easier to perform." b) "There's no other option but to assume the knee-chest position." c) "Lying on your left side will be fine during the procedure." d) "I'll report your concerns to the physician."

A

Knowing Maslow's hierarchy of needs can assist a nurse in understanding a client's behavior. Place the stages of Maslow's hierarchy of needs in order from basic to most complex.

1) Physiologic Needs 2) Safety and Security 3) Love and Belonging 4) Self Esteem 5) Self-Actualization

A client experiencing a manic phase of bipolar disorder sustained cuts on the body from falling through a store window. The nurse is preparing to start an intravenous needle insertion and explains the procedure to the client a) Using clear and simple terms b) Standing next to the bed with arms crossed c) Ignoring the client's statement, "I don't want this" d) Interrupting the client's ravings

A

A client in an inpatient psychiatric unit tells the nurse, "I'm going to divorce my no-good husband. I hope he rots in hell. But I miss him so bad. I love him. When's he going to come get me out of here?" The nurse interprets the client's statements as indicative of which of the following? a) Ambivalence. b) Autistic thinking. c) Associative looseness. d) Auditory hallucinations.

A

A client is 2 months pregnant. Which factor should the nurse anticipate as most likely to affect her psychosocial transition during pregnancy? a) Support from her partner b) Readiness at home for the baby c) Previous health promotion activities d) The month of her due date

A

A client is hospitalized with a traumatic brain injury following an automobile accident. The client has difficulty processing information and needs information to be repeated. A consulting physician enters the room. The nurse a) Turns off the television b) Removes unnecessary items from the beside table while the physican meets with the client c) Communicates with the daughter while the physician talks with the client d) Leaves the room

A

A client reports to the nurse that her grandmother with Alzheimer's disease recently moved in with her and her two school-aged children. The client states the grandmother becomes agitated and starts yelling and crying frequently. The woman asks, "What can I do?" The nurse first responds: a) "What precipitates the outbursts?" b) "Start rubbing her shoulders and her back." c) "You need to remain calm during the outbursts." d) "Play quiet music that your grandmother may like."

A

A client who is paraplegic cannot feel the lower extremities and has been positioned on the side. The nurse should inspect which of the following areas that is a potential pressure point when the client is in this position? a) Ankles. b) Sacrum. c) Heel. d) Occiput.

A

A client with schizophrenia is having an acute exacerbation of symptoms. The client states, "Black cats and black hats. Where does the time go?" Which of the following would be most important for the nurse to say? a) "What's the connection between cats, hats, and time?" b) "Halloween is getting close, isn't it." c) "Do you have a black cat?" d) "Time certainly does go faster these days."

A

A home care nurse visits an older adult client with dementia due to Alzheimer's disease. As a result of the client's confused thinking, the client is experiencing significant difficulty in communicating with family members. Which intervention would be most appropriate for this client? a) Validate the client's current needs b) Orient the client to reality c) Provide appropriate sensory appliances d) Maintain levels of sensory stimulation

A

A nurse is caring for a client who has limited mobility and requires a wheelchair. The nurse has concern for circulation problems when which device is used? a) Ring or donut b) Specialty mattress c) Gel flotation pad d) Water bed

A

A nurse is caring for an elderly client at risk of injury due to confusion. The client has a stable gait. Which of the following methods of restraining should the nurse use? a) Alarm-activating bracelet b) Locking lap tray chair c) Waist restraint d) Vest restraint

A

A nurse is caring for an elderly client who has been confined to bed for several weeks following a fall. The client has been exhibiting symptoms of sundowner's syndrome. Which of the following are characteristics of sundowner's syndrome? a) Feeling agitated and wakeful at night b) Awakening more frequently c) Requiring longer time to fall asleep d) Napping during the daytime

A

A nurse is performing a head and neck assessment of a patient suspected of having leukemia. How would the nurse detect enlarged lymph nodes commonly associated with this disease? a) Inspect and palpate the supraclavicular area. b) Inspect and palpate the left and then the right carotid arteries. c) Palpate the thyroid gland. d) Inspect the patient's ability to move his or her neck.

A

A nurse is performing a preoperative assessment. Which client statement should alert her to the presence of risk factors for postoperative complications? a) "I've cut my smoking down from two packs to one pack per day." b) "I haven't been able to eat anything solid for the past 2 days." c) "I had an operation 2 years ago, and I don't want to have another one." d) "I've never had surgery before."

A

A nurse is reviewing a patient's laboratory test results. Which serum albumin level would lead the nurse to suspect that the patient is at risk for pressure ulcers? a) 2.5 g/mL b) 3.1 g/mL c) 3.5 g/mL d) 4.0 g/mL

A

A nurse measures a client's apical pulse rate and compares it with his radial pulse rate. The differential between these two pulses is called: a) the pulse deficit. b) pulsus regularis. c) the pulse pressure. d) the pulse rhythm.

A

A patient informs the nurse that she is not able to recall her phone number or address, and this is concerning to the patient. The nurse recognizes that the inability to recall information is indicative of which sensory/perception problem? a) Impaired memory b) Chronic confusion c) Acute confusion d) Disturbed sensory perception

A

A patient is diagnosed with mild dementia while in the hospital. In preparing for discharge, the nurse should discuss with the family the: a) Possible need for home care b) Lack of free resources of care c) Legal responsibility for the future d) Need for transfer to a long-term care facility

A

A patient with brain tumor is undergoing radiation and chemotherapy for treatment of cancer. Of late, the patient is complaining of swelling in the gums, tongue, and lips. Which of the following is the most likely cause of these symptoms? a) Stomatitis b) Neutropenia c) Extravasation d) Nadir

A

A physician orders the application of a warm, sterile compress to reduce edema in a patient's wound. Which of the following is a recommended step in this procedure? a) Keep the dressing in place for the prescribed amount of time or up to 30 minutes. b) Cover the site with a three layers of gauze and with a clean dry bath towel. c) Place an aquathermia or heating device directly on the dressing. d) Apply pressure to the compress to mold it around the wound site.

A

After total hip replacement, a client is receiving epidural analgesia to relieve pain. Which action is a nursing priority for this client? a) Assessing for sensation in the legs b) Changing the catheter site dressing every shift c) Assessing capillary refill time d) Keeping the client flat in bed

A

An 18-year-old is highly dependent on her parents and fears leaving home to attend college. Shortly before the fall semester starts, she complains that her legs are paralyzed and is rushed to the emergency department. When physical examination rules out a physical cause for her paralysis, the physician admits the woman to the psychiatric unit, where she is diagnosed with functional neurologic symptom disorder. She asks the nurse, "Why has this happened to me?" What is the nurse's best response? a) "Your problem is real but, there is no physical basis for it. We'll work on what is going on in your life to find out why it's happened." b) "It's common for someone with your personality to develop a conversion disorder during times of stress." c) "You've developed this paralysis so you will have a reason to stay with your parents. You must deal with this conflict if you want to walk again." d) "It must be awful not to be able to move your legs. You may feel better if you realize the problem is psychological, not physical."

A

An advantage of using biologic burn grafts such as porcine (pigskin) grafts is that they: a) Promote the growth of epithelial tissue. b) Provide for permanent wound closure. c) Encourage formation of tough skin. d) Facilitate development of subcutaneous tissue.

A

An elderly client admitted with pneumonia and dementia has attempted several times to pull out the IV and Foley catheter. The nurse obtains a prescription for bilateral soft wrist restraints. Which nursing action is most appropriate? a) Attach the ties of the restraints to the bedframe. b) Perform circulation checks to bilateral upper extremities each shift. c) Reevaluate the need for restraints and document weekly. d) Ensure the restraint order has been signed by the physician within 72 hours.

A

An older adult patient enjoys good overall health, but has just been diagnosed with pneumonia and has begun receiving an intravenous (IV) antibiotic. Shortly after being administered the first dose, the patient pulled out his IV line and is now attempting to scale his bedrails. Which of the following phenomena most likely underlies this change in the patient's cognition? a) Delirium b) Dementia c) Disorientation d) Depression

A

During the nurse's morning assessment of a patient with a diagnosis of dementia, the patient states that the year is 1949 and she believes she is in a hotel. How should the nurse best respond to this patient's disorientation? a) Reorient the patient to place and time. b) Thank the patient for her responses and document her cognitive status. c) Ask the patient what she was doing in 1949 and what hotel she believes she is in. d) Provide hints during conversation as to the correct year and place.

A

Following a laryngectomy, the nurse notices that the client has saliva collecting beneath the skin flaps. This finding is indicative of which of the following? a) Development of a fistula. b) Skin necrosis. c) Carotid artery rupture. d) Stomal stenosis.

A

In myasthenia gravis (MG), there is a decrease in the number of receptor sites of which neurotransmitter? a) Acetylcholine b) Norepinephrine c) Epinephrine d) Dopamine

A

Pet therapy is commonly used in long-term facilities for distraction. If a patient is experiencing pain and the pain is temporarily decreased while petting a visiting dog or cat, this is an example of which type of distraction technique for patients experiencing pain? a) Tactile kinesthetic distraction b) Auditory distraction c) Visual distraction d) Project distraction

A

The client with acute renal failure is recovering and asks the nurse, "Will my kidneys ever function normally again?" The nurse's response is based on knowledge that the client's renal status will most likely: a) Continue to improve over a period of weeks. b) Improve only if the client receives a renal transplant. c) Result in end-stage renal failure. d) Result in the need for permanent hemodialysis.

A

The client with dual diagnoses of major depression and alcohol abuse states, "I only drink when I can't sleep." Which outcome is important for the client to achieve first? a) Verbalize the desire to stop drinking alcohol. b) Describe dangerous effects when combining alcohol and antidepressant medication. c) Verbalize negative effects of alcohol on the body. d) Describe adaptive methods of coping to induce sleep.

A

The mother of a newborn is voicing concerns about her baby's ability to hear. The nurse should tell the mother: a) Most American states and Canadian jurisdictions mandate hearing tests for infants. b) She can test the baby's hearing by clapping her hands 24 inches (60 cm) from the infant's head. c) Her concern is unfounded because hearing problems are rare in newborns. d) Newborns cannot hear well until they are at least 6 weeks old.

A

The nurse has identified the following nursing diagnosis on the plan of care for a client who has undergone breast cancer surgery: Disturbed sensory perception related to nerve irritation in the affected breast area. Which of the following indicates that the outcome has been achieved? a) Client states that feeling in her breast area will gradually subside with time. b) Client demonstrates appropriate use of prescribed analgesic. c) Client reports a sensation of pulling in the breast area. d) Client identifies appropriate measures to reduce the risk of lymphedema.

A

The nurse is caring for a client recovering from a major burn. Burns affect the immune system by causing a loss of large amounts of which of the following? a) Serum, which depletes the body's store of immunoglobulins b) Plasma, which depletes the body's store of catecholamines c) Serum, which depletes the body's store of glucagon d) Plasma, which depletes the body's store of calcitonin

A

The nurse is employed in a neurologist's office, performing a history and assessment on a client experiencing hearing difficulty. The nurse is most correct to gather equipment to assess the function of which cranial nerve? a) Cranial nerve VIII b) Cranial nerve XI c) Cranial nerve VI d) Cranial nerve II

A

The nurse is evaluating a skin lesion on a client brought to the emergency department. The nurse notes characteristics of chickenpox but has the physician evaluate the lesion for which biologic disaster agent? a) Smallpox b) Botulism c) Anthrax d) Rubella

A

The nurse is performing an initial nursing assessment on a client with possible Guillain-Barre syndrome. Which of the following findings would be most consistent with this diagnosis? a) Muscle weakness and hyporeflexia of the lower extremities b) Ptosis and muscle weakness of upper extremities c) Fever and cough d) Hyporeflexia and skin rash

A

The nurse's morning assessment of a patient who has a history of heart failure reveals the presence of 2+ pitting edema in the patient's ankles and feet bilaterally. What is this assessment finding suggestive of? a) Fluid volume excess b) Hypovolemia c) Metabolic acidosis d) Hyponatremia

A

The nursing instructor informs a student nurse that a patient she is caring for has a chronic neurologic condition and the condition decreases the patient's peristalsis. What nursing diagnosis is the most likely risk for this patient? a) Constipation b) Diarrhea c) Excessive fluid volume d) Deficient fluid volume

A

The plan of care for a patient exhibiting signs of sensory deprivation includes incorporating tactile stimulation. Which nursing intervention will provide tactile stimulation? a) Providing a backrub with morning and evening care b) Orienting the patient to his environment c) Delivering meticulous oral care d) Placing a calendar and clock on the patient's bedside table

A

What patient activity indicates to the nurse that a patient understands how to correctly instill ophthalmic medications? a) Client pulls the tissue near the cheek downward to instill medication. b) Client allows the tip of the container to touch the eyelid while administering the medication. c) Client wipes the lids and lashes prior to instillation in a direction toward the nose with moistened, soft gauze. d) Client rubs the eye after administering medication.

A

When a client with functional neurologic symptom disorder reports blindness, ophthalmologic examinations reveal that no physiologic disorder is causing progressive vision loss. The most likely source of this client's reported blindness is: a) having been forced to watch the torture of a loved one. b) daily use of antianxiety agents and alcoholic beverages. c) a family history of major depression. d) noncompliance with a psychotropic medication regimen.

A

When caring for a patient with advanced cirrhosis and hepatic encephalopathy, which of the following assessment findings should the nurse report immediately? a) Change in the patient's handwriting and or cognitive performance b) Weight loss of 2 pounds in 3 days c) Constipation for more than 2 days d) Anorexia for more than 3 days

A

When performing a physical assessment of a client, the client reports numbness, tingling, and pain when the nurse percusses lightly over the median nerve. The nurse recognizes that this finding is consistent with: a) Carpal tunnel syndrome b) Impingement syndrome c) Morton's neuroma d) Dupuytren's contracture

A

Which nursing activity supports the principles of palliative care for a dying infant and his family? a) Creating a therapeutic, homelike environment for the infant and his family b) Clustering care activities to provide as much rest as possible for the infant c) Minimizing noise and disruption to decrease stress for the infant d) Maintaining routines and structure for the infant and his family

A

Which of the following allows each person to experience the environment differently? a) Perception b) Growth c) Chromosomes d) Socioeconomics

A

Which of the following is a term used to describe a soft tissue injury produced by a blunt force? a) Contusion b) Strain c) Hematoma d) Sprain

A

Which of the following techniques is used to surgically revascularize the myocardium? a) Minimally invasive direct coronary bypass b) Balloon bypass c) Peripheral bypass d) Gastric bypass

A

Which of the following terms describes an opening between the bladder and the vagina? a) Vesicovaginal fistula b) Rectocele c) Cystocele d) Rectovaginal fistula

A

Which of the following terms refers to a graft derived from one part of a patient's body and used on another part of that same patient's body? a) Autograft b) Heterograft c) Homograft d) Allograft

A

Which terms refers to the progressive hearing loss associated with aging? a) Presbycusis b) Sensorineural hearing loss c) Otalgia d) Exostoses

A

Which type of fracture is one in which the skin or mucous membranes extends to the fractured bone? a) Compound b) Simple c) Complete d) Incomplete

A A compound fracture is one in which the skin or mucous membrane wound extends to the fractured bone. A complete fracture involves a creak across the entire cross-section of the bone and is frequently displaced. An incomplete fracture involves a break through only part of the cross-section of the bone. A simple fracture is one that does not cause a break in the skin.

A client returns from a myelogram, for which an iodized oil was used. Which of the following nursing measures should be included in the care of this client? a) Assessment of lower extremity movement and sensation. b) Restricted fluid intake. c) Bed rest with bathroom privileges. d) Head of the bed elevated 45 degrees.

A Neurologic status in the lower extremities is assessed frequently, as is the client's ability to void. This is done to determine if there is any nerve impairment. *Regardless of the type of dye used for the test, bed rest is required for several hours after a myelogram.* Fluid intake is encouraged to replace cerebrospinal fluid, to reduce headache, and to facilitate absorption of retained contrast medium. Nursing care of the client after a myelogram depends in part on the type of dye used. For example, if an oil contrast was used, the client will usually lie flat for 8 to 12 hours. If a water-soluble contrast was used, the head of the bed is elevated 45 degrees for 8 to 24 hours. This position reduces the rate of upward dispersion of the contrast medium.

Which of the following clinical manifestations of type 2 diabetes occurs if glucose levels are very high? a) Oliguria b) Increased energy c) Hyperactivity d) Blurred vision

D

The nurse is providing discharge instructions to the client with peripheral vascular disease. Which of the following instructions should be included in the discussion with this client? Select all that apply. a) Keeping extremities elevated on pillows. b) Avoiding prolonged standing and sitting. c) Using a heating pad to promote vasodilation. d) Keeping the legs in a dependent position. e) Limiting walking so as not to activate the "muscle pump."

A, B

During a home visit to an elderly client with mild dementia, the client's daughter reports that she has one major problem with her mother. She says, "She sleeps most of the day and is up most of the night. I can't get a decent night's sleep anymore." Which suggestions should the nurse make to the daughter? Select all that apply. a) Establish a set routine for rising, hygiene, meals, short rest periods, and bedtime. b) Engage the client in simple, brief exercises or a short walk when she gets drowsy during the day. c) Promote relaxation before bedtime with a warm bath or relaxing music. d) Ask the client's primary health care provider for a strong sleep medicine. e) Have the daughter encourage the use of caffeinated beverages during the day to keep her mother awake.

A, B, C

The nurse is caring for a 5-year-old child who is cognitively challenged. The parents ask the nurse how best to foster independence in the child. Which of the following teaching points should the nurse emphasize? Select all that apply. a) Use repetition to reinforce learning. b) Teach one step at a time to facilitate short-term memory. c) Use generous praise as a reward for learning. d) Teach your child with a group of other children. e) Limit principles and abstract concepts in the teaching.

A, B, C, E

Which of the following should the nurse include in the discharge plan for a client with multiple sclerosis who has an impaired peripheral sensation? Select all that apply. a) Carefully test the temperature of bath water. b) Wear warm clothing when outside in cold temperatures. c) Avoid hot water bottles and heating pads. d) Avoid kitchen activities because of the risk of injury. e) Inspect the skin daily for injury or pressure points.

A, B, C, E

A patient has been brought to the emergency room after being hit in the head with a baseball. The nurse should be alert to which of the following clinical manifestations of a detached retina? Select all that apply. a) Bright flashing lights b) Sensation of a curtain coming across vision of one eye c) Pain d) Sudden onset of floaters e) Cobwebs

A, B, D, E

A client who was transferred from a long-term care facility is admitted with dehydration and pneumonia. Which nursing interventions can help prevent pressure ulcer formation in this client? Select all that apply. a) Perform range-of-motion exercises. b) Use commercial soaps to keep the skin dry. c) Encourage the client to eat a well-balanced diet. d) Reposition the client every 2 hours. e) Tuck bed covers tightly into the foot of the bed.

A, C, D

Today you are caring for Mr. Cantrell, a 69-year-old patient. He has gradually lost much of the ability to hear in both ears due to working with loud machinery all of his working life. Which of the following interventions will you add to Mr. Cantrell's care plan in order to make him more comfortable with his hearing loss? Choose all that apply. a) Do not chew gum or food when speaking b) Avoid verbal conversation when possible c) Face the patient; use meaningful gestures d) Decrease background noise if possible e) Be aware of nonverbal communication

A, C, D, E

You are teaching a group of patients about general eye care to prevent vision loss and eye injury. Which of the following will you include in your presentation? Choose all that apply. a) Use caution with corrosive agents b) Use a saline eye rinse daily c) Avoid eye strain and rubbing eyes d) Wear protective goggles for mowing lawns e) Avoid eye damage from ultraviolet rays

A, C, D, E

The client with blindness is hospitalized following a myocardial infarction. Which of the following care measures would the nurse take with this client? Select all answers that apply. a) Identify self when walking into the client's room. b) Pat the canine service animal immediately upon entering the client's room. c) State when the nurse is leaving the room. d) Speak in a slightly louder voice. e) Leave the bathroom door either completely open or closed.

A, C, E

A client comes to the clinic for diagnostic allergy testing. The nurse understands that intradermal injections are used for such testing based on which principle? a) Intradermal drugs have fast onset of action. b) Intradermal drugs diffuse more slowly. c) Intradermal drugs are easier to administer. d) Intradermal injection is less painful.

B

A patient brought to the emergency room is unconscious and cannot be aroused. The patient is breathing and has a heartbeat. What state of awareness is this patient exhibiting? a) Stupor b) Somnolence c) Coma d) Asleep

C

The nurse teaches the mother of a toddler who has had cleft palate repair that her child is at risk for developing which of the following in the future? a) Hearing problems. b) Poor self-concept. c) A speech defect. d) Chronic sinus infections.

C

A client complains of sporadic epigastric pain, yellow skin, nausea, vomiting, weight loss, and fatigue. Suspecting gallbladder disease, the physician orders a diagnostic workup, which reveals gallbladder cancer. Which nursing diagnosis is appropriate for this client? a) Chronic low self-esteem b) Anticipatory grieving c) Impaired swallowing d) Disturbed body image

B

A client has been diagnosed with colon cancer with metastasis to the lymph nodes. When the nurse enters the room, the client says life is "not worth living." What is the nurse's best therapeutic response? a) Assure the client that everything will work out fine. b) Approach the client and ask if there are questions about the condition. c) Ask the client if calling the family would be helpful. d) Explain that the condition is complicated and ask a physician to come speak with the client.

B

A client has severe arterial occlusive disease and gangrene of the left great toe. Which of the following findings is expected? a) Thin, soft toenails. b) Loss of hair on the lower leg. c) Edema around the ankle. d) Warmth in the foot.

B

A client in the final stages of terminal cancer tells his nurse: "I wish I could just be allowed to die. I'm tired of fighting this illness. I have lived a good life. I continue my chemotherapy and radiation treatments only because my family wants me to." What is the nurse's best response? a) "I know you are tired of fighting this illness, but death will come in due time." b) "Would you like to meet with your family and your physician about this matter?" c) "Would you like to talk with your minister about the significance of death?" d) "Would you like to talk with a psychologist about your thoughts and feelings?"

B

A client is admitted with glomerulonephritis. In addition to physical problems, which other psychosocial problems could likely affect this client? a) Altered sexuality patterns related to prolonged hospitalization b) Anxiety related to poorly functioning kidneys and body image disturbance c) Altered breathing patterns related to dehydration d) Impaired physical mobility related to bone reabsorption

B

A client periodically has acute panic attacks. These attacks are unpredictable and have no apparent association with a specific object or situation. During an acute panic attack, the client may experience: a) a decreased heart rate. b) a decreased perceptual field. c) heightened concentration. d) a decreased respiratory rate.

B

A client reports that she has gained weight and that her face and body are "rounder," while her legs and arms have become thinner. A tentative diagnosis of Cushing's disease is made. When examining this client, the nurse would expect to find: a) Muscle hypertrophy in the extremities. b) Bruised areas on the skin. c) Decreased body hair. d) Orthostatic hypotension.

B

A client seeks care for lower back pain of 2 weeks' duration. Which assessment finding suggests a herniated intervertebral disk? a) Back pain when the knees are flexed b) Pain radiating down the posterior thigh c) Homans' sign d) Atrophy of the lower leg muscles

B

A client tells a nurse that he has a rash on his back and right flank. The nurse observes elevated, round, blisterlike lesions filled with clear fluid. When documenting the findings, what medical term should the nurse use to describe these lesions? a) Papules b) Vesicles c) Pustules d) Plaque

B

A client with a penile implant is to be informed about the possible complications after discharge. Which of the following are complications that occur due to a midsized implant, pressure, or friction of the implanted cylinders? a) Erosion of scrotal, bowel, or bladder tissue b) Erosion of penile or urethral tissue c) Malfunction of the device d) Migration of the cylinders, pump, and reservoir

B

A client with hemolysis, elevated liver enzymes, and low platelet count (HELLP) syndrome is admitted to the labor and delivery unit. The client's condition rapidly deteriorates and despite efforts by the staff, the client dies. After the client's death, the nursing staff displays many emotions. With whom should the nurse-manager consult to help the staff cope with this unexpected death? a) The social worker, so she can contact the family about funeral arrangements and pass along the information to the nursing staff b) The chaplain, because his educational background includes strategies for handling grief c) The physician, so he can provide education about HELLP syndrome d) The human resource director, so she can arrange vacation time for the staff

B

A client with severe peptic ulcer disease has undergone surgery and is several hours postoperative. During assessment, the nurse notes that the client has developed cool skin, tachycardia, and labored breathing; the client also appears to be confused. Which of the following complications has the client most likely developed? a) Perforation b) Hemorrhage c) Pyloric obstruction d) Penetration

B

A community health nurse teaches a group of seniors about modifiable risk factors that contribute to the development of peripheral arterial disease (PAD). The nurse knows that the teaching was effective based on which of the following statements? a) "The older I get the higher my risk for peripheral arterial disease gets." b) "I will need to stop smoking because the nicotine causes less blood to flow to my hands and feet." c) "I will need to increase the amount of green leafy vegetables I eat to lower my cholesterol levels." d) "Since my family is from Italy, I have a higher risk of developing peripheral arterial disease."

B

A medicalsurgical nurse is assisting a wound care nurse with the debridement of a patient's coccyx wound. What is the primary goal of these nurses' action? a) Removing excess drainage and wet tissue to prevent maceration of surrounding skin b) Removing dead or infected tissue to promote wound healing c) Removing purulent drainage from the wound bed in order to accurately assess it d) Stimulating the wound bed to promote the growth of granulation tissue

B

A nurse is assessing an immobile client and notes an area of sacral skin is reddened, but not broken. The reddened area continues to blanch and refill with fingertip pressure. The most appropriate nursing action at this time is to: a) Apply a moist-to-moist dressing, being careful to pack just the wound bed. b) Reposition the client off the reddened skin and re-assess in a few hours. c) Consult with a wound-ostomy-continence nurse specialist. d) Complete and document a Braden skin breakdown risk score for the client.

B

A nurse is assisting a patient to shave his beard. Which of the following statements accurately describes a recommended step in this process? a) Fill a basin with cool water. b) Apply cream to area to be shaved in a layer about ½-inch thick. c) Shave against the direction of hair growth in upward, short strokes d) Cover the patient with a blanket.

B

A nurse is caring for a client admitted with a diagnosis of exacerbation of myasthenia gravis. Upon assessment of the client, the nurse notes the client has severely depressed respirations. The nurse would expect to identify which acid-base disturbance? a) Respiratory alkalosis b) Respiratory acidosis c) Metabolic alkalosis d) Metabolic acidosis

B

A nurse is dressing the wound of a client who is admitted to the outpatient surgical unit. Which of the following is a major advantage of outpatient surgery? a) It requires intensive pre-operative teaching in a short time. b) It interferes less with the client's daily routine. c) It reduces the time for establishing a nurse-client rapport. d) It allows less opportunity for family contact and support.

B

A nurse is providing care to a client who was a victim of a blast injury that occurred in an enclosed building. The nurse assesses the client closely for which of the following as occurring most frequently? a) Internal organ damage b) Tympanic membrane rupture c) Bone fractures d) Head injuries

B

A nurse is teaching a client with a leg ulcer about tissue repair and wound healing. Which statement by the client indicates understanding? a) "I'll make sure that I keep the site covered at all times." b) "I'll eat plenty of fruits and vegetables." c) "I will limit my intake of red meat to once a week." d) "Increase in redness of the ulcer means better blood flow."

B

A patient has an elevated serum ammonia level and is exhibiting mental status changes. The nurse should suspect which of the following conditions? a) Portal hypertension b) Hepatic encephalopathy c) Cirrhosis d) Asterixis

B

A rehabilitation nurse is assisting a patient to cope with a disability. Which of the following would the nurse suggest? a) Stop any activity once fatigue occurs. b) Emphasize areas of strengths. c) Group any heavy work to be done at the same time. d) Avoid seeking help from others.

B

A single parent, aged 17 years with one child and pregnant with her second child, has the mental age of a 12-year-old. As the home care nurse, what is your greatest concern in caring for this woman? a) Her ability to receive financial aid b) Her cognitive ability to understand c) Her physical care abilities d) Her ability to bond with her children

B

A spinal cord injury patient has no awareness of the need to void. This type of incontinence is termed a) toilet incontinence. b) reflex (neurogenic) incontinence. c) functional incontinence. d) stress incontinence.

B

Crackles heard on lung auscultation indicate which of the following? a) Bronchospasm. b) Fluid-filled alveoli. c) Cyanosis. d) Airway narrowing.

B

Following a percutaneous transluminal coronary angioplasty (PTCA), which of the following medications classifications would be used to prevent thrombus formation in the stent? a) Calcium channel blockers b) Antiplatelets c) Nitrates d) Beta blockers

B

High doses of this medication can produce bilateral tinnitus? a) Dramamine b) Aspirin c) Promethazine d) Antivert

B

If a client has abdominal surgery and a portion of the small intestine is removed, the client is at risk for which of the following? a) Constipation b) Malabsorption syndrome c) Gastric ulcers d) Cirrhosis

B

If a dislocation is not treated promptly, tissue death due to anoxia can occur. This would be documented as which of the following? a) Heterotopic ossification b) Avascular necrosis (AVN) c) Osteomyelitis d) Subluxation

B

Matt Thompson, a 37-year-old farmer, has been diagnosed with pre diabetes. Following his visit with his primary care provider, you begin your client education session to discuss treatment strategies. What can be the consequences of untreated pre diabetes? a) Type 2 diabetes b) All options are correct. c) CVA d) Cardiac disease

B

Mr. Fields is a resident of a long-term care facility who has moderate hearing loss. When communicating with Mr. Fields, what should the nurse do? a) Use vocabulary and concepts that are as simple and unambiguous as possible. b) Minimize background noises and ensure that lighting is adequate to see the nurse's face. c) Use written communication whenever possible in order to minimize Mr. Fields' frustration. d) Repeat each direction or question in different terms in order to maximize understanding.

B

One of the patients you are caring for today is Jeff, a 13-year-old boy who has suffered a concussion while playing hockey. Your morning assessment finds him very drowsy but he responds normally to stimuli. You document his level of consciousness as which of the following? a) Stupor b) Somnolence c) Coma d) Asleep

B

Prevention of skin breakdown and maintenance of skin integrity among older clients is important because they are at greater risk secondary to: a) Impaired visual acuity. b) Altered protective pressure sensation. c) Altered balance. d) Impaired hearing ability.

B

Six hours after undergoing an abdominal hysterectomy, a client has a strong urge to void and voids 25 ml into the bedpan. Based on these data, the nurse determines that the client: a) Has developed a urinary tract infection and needs antibiotics. b) Is experiencing urine retention and needs to be catheterized. c) Needs more time to try to void and tells the client to try again in 1 hour. d) Is probably dehydrated and needs additional intravenous fluids.

B

The acute care nurse is preparing to care for an 86-year-old patient who just returned to the unit after surgery to repair a fractured hip. The patient has severe dementia. Which of the following pain management strategies would be most appropriate for this patient? a) Patient controlled analgesia (PCA) b) Authorized agent-controlled analgesia (AACA) c) As needed (PRN) administration of intramuscular analgesic medications d) As needed (PRN) administration of oral analgesic medications

B

The nurse asks a client to point to where she feels pain. The client asks why this is important. The nurse's best response would be which of the following? a) "This determines the pain medication to be ordered." b) "Often the area of pain is referred from another area." c) "The area may determine the severity of the pain." d) "If the doctor massages over the exact painful area, the pain will disappear."

B

The nurse is assessing a 60-year-old male who has hoarseness. The nurse should conduct a focused assessment to determine: a) Exposure to sun and family history of head and neck cancers. b) History of tobacco use and alcohol consumption. c) Patterns of medication use and history of alcohol consumption. d) Exposure to wood dust and a high-fat diet.

B

The nurse is assessing a male patient with a diagnosis of vascular dementia. As a result of his cognitive deficit, the patient is unable to provide many of the data that are required on the hospital's nursing admission history document. How should the nurse best proceed with this assessment? a) Limit the assessment to objective data. b) Supplement the patient's information by speaking with family or friends. c) Perform the assessment in several short episodes rather than at one sitting. d) Obtain the patient's records from admissions to other institutions.

B

The nurse is caring for an elderly client who has experienced a sensorineural hearing loss. The nurse anticipates that the client will exhibit which one of the following symptoms? a) Inability to assign meaning to sound. b) Difficulty hearing high-pitched sounds. c) Problems with speaking clearly. d) Vertigo when changing positions.

B

The nurse should assess an older adult with macular degeneration for: a) Loss of peripheral vision. b) Loss of central vision. c) Total blindness. d) Blurring of vision.

B

The nurse should use extreme caution when applying heat therapy to which of the following patients? a) A patient with high pain sensitivity b) A patient who is unconscious c) A patient who is receiving corticosteroids d) A patient with a venous ulcer

B

The nurse would expect which of the following diagnostic tests to be ordered for a patient with lower extremity muscle weakness? a) Arthrocentesis b) Electromyograph (EMG) c) Bone scan d) Biopsy

B

The priority nursing diagnosis for a client who has just been admitted to the hospital with burns would be which of the following? a) Impaired social interaction b) Impaired skin integrity c) Body image disturbance d) Risk for altered nutrition

B

To encourage adequate nutritional intake for a client with Alzheimer's disease, a nurse should: a) fill out the menu for the client. b) stay with the client and encourage him to eat. c) help the client fill out his menu. d) give the client privacy during meals.

B

To reduce the risk of pressure ulcer formation, which of the following activities should the nurse teach the client who is wheelchair-bound as a result of a spinal cord injury? a) Eat a high-carbohydrate diet. b) Shift your weight every 15 minutes. c) Move from the bed to the wheelchair every 2 hours. d) Bathe daily.

B

Upon assessment of a patient with myasthenia gravis, the nurse observes drooping of the upper eyelids. What is this finding is known as? a) Miosis b) Ptosis c) Entropion d) Ectropion

B

When developing a care plan for a client newly diagnosed with scleroderma, which nursing diagnosis has the highest priority? a) Impaired gas exchange b) Impaired skin integrity c) Risk for constipation d) Imbalanced nutrition: More than body requirements

B

Which of the following complications is common for victims of electrical burns? a) Hypovolemic shock b) Cardiac dysrhythmia c) Infection d) Inhalation injury

B

Which of the following goals is a priority for a client with rheumatoid arthritis? The client will: a) Verbalize that recovery from rheumatoid arthritis will require several years of treatment. b) Demonstrate use of adaptive equipment. c) Minimize the frequency with which anti-inflammatory drugs are used to control joint discomfort. d) Learn to limit activity so as to avoid joint pain.

B

Which of the following is a true statement regarding hospice care? a) Patients have an acute illness b) Patients have a life expectancy of 6 months or less c) It encourages the prolongation of life through artificial means d) It is cure-focused

B

Which of the following medications is the most effective agent in the treatment of Parkinson's disease (PD)? a) Bromocriptine mesylate (Parlodel) b) Levodopa (Larodopa) c) Benztropine (Cogentin) d) Amantadine (Symmetrel)

B

The wound care clinical nurse specialist has been consulted to evaluate a wound on the leg of a patient with diabetes. The wound care nurse determines that damage has occurred to the subcutaneous tissues; how would she document this wound? a) Stage II pressure ulcer b) Stage III pressure ulcer c) Stage IV pressure ulcer d) Stage I pressure ulcer

B Damage to the subcutaneous tissue indicates a stage III ulcer. Extensive destruction associated with full-thickness skin loss is categorized as a stage IV pressure ulcer. A stage I ulcer is a defined area of persistent redness in lightly pigmented skin and a persistent red, blue, or purple hue in darker pigmented skin. A stage II pressure ulcer is superficial and may present as a blister or abrasion.

A patient has been diagnosed as having global aphasia. The nurse recognizes that the patient will be unable to do which of the following actions? a) Form understandable words b) Form understandable words and comprehend the spoken word c) Speak at all d) Comprehend the spoken word

B Global aphasia is a combination of expressive and receptive aphasia and presents tremendous challenge to the nurse to communicate effectively with the patient. In receptive aphasia, the patient is unable to form words that are understandable. In expressive aphasia, the patient is unable to form words that are understandable. The patient who is unable to speak at all is referred to as mute.

If untreated, squamous cell carcinoma of the external ear can spread through the temporal bone causing what effect? a) Nystagmus b) Facial nerve paralysis c) Diplopia d) Motor impairment

B If untreated, squamous cell carcinomas of the ear can spread through the temporal bone, causing facial nerve paralysis and hearing loss.

A client with schizophrenia tells the nurse, "My intestines are rotted from the worms chewing on them." This statement indicates a: a) delusion of grandeur. b) somatic delusion. c) jealous delusion. d) delusion of persecution.

B Somatic delusions focus on bodily functions or systems and commonly include delusions about foul odor emissions, insect infestations, internal parasites, and misshapen parts. Delusions of persecution are morbid beliefs that one is being mistreated and harassed by unidentified enemies. Delusions of grandeur are gross exaggerations of one's importance, wealth, power, or talents. Jealous delusions are delusions that one's spouse or lover is unfaithful.

To evaluate a client's atrial depolarization, the nurse observes which part of the electrocardiogram waveform? a) PR interval b) P wave c) QRS complex d) T wave

B The P wave depicts atrial depolarization, or spread of the electrical impulse from the sinoatrial node through the atria. The PR interval represents spread of the impulse through the interatrial and internodal fibers, atrioventricular node, bundle of His, and Purkinje fibers. The QRS complex represents ventricular depolarization. The T wave depicts the relative refractory period, representing ventricular repolarization.

A nurse correctly identifies which items as belonging to the dorsal cavity? a) Mediastinum b) Vertebral canal c) Mouth d) Reproductive organs

B The dorsal cavity consists of the cranial (skull) and vertebral canal (spinal cavity). The mediastinum and reproductive organs are located in the ventral cavity. The mouth is located in the oral cavity. (less)

A nurse inspecting a patient's pressure ulcer documents the following: full-thickness tissue loss; visible subcutaneous fat; bone, tendon, and muscle are not exposed. This pressure ulcer is categorized to be at which of the following stages? a) Stage IV b) Stage III c) Stage II d) Stage I

B n stage III there is full-thickness tissue loss; subcutaneous fat may be visible, but bone, tendon, or muscle are not exposed. In stage I there is intact skin with nonblanchable redness of a localized area, usually over a bony prominence. In stage II there is partial thickness loss of dermis presenting as a shallow open ulcer with a red-pink wound bed, without slough, and in stage IV, there is full-thickness tissue loss with exposed bone, tendon, or muscle.

a) Patients who are taking multiple medications b) Patients whose primary language is different from that of the healthcare team c) Infants d) Young children e) Patients who have significant visual impairment f) Patients who have dementia

B, C, D, F

Mr. Quinn has just been diagnosed with osteomyelitis. Osteomyelitis is an infection of the bone, resulting in limited blood supply to the bone, inflammation of and pressure on the tissue, bone necrosis, and formation of new bone around devitalized bone tissue. What are possible causes of osteomyelitis? Select all that apply. a) Progressive osteoporosis b) Trauma, such as penetrating wounds or compound fractures c) Surgical contamination, such as pin sites of skeletal traction d) Vascular insufficiency in clients with diabetes or peripheral vascular disease

B, D

The family of a client, diagnosed with Alzheimer's disease, wants to keep the client at home. They say that they have the most difficulty in managing his wandering. The nurse should instruct the family to do which of the following? (Select all that apply). a) Ask the primary health care provider for a sleeping medication. b) Install motion and sound detectors. c) Have a relative sit with the client all night. d) Install door alarms and high door locks. e) Have the client wear a Medical Alert bracelet.

B, D, E

The stigma related to having a mental illness, especially a chronic illness, persists despite improvements in the management of illnesses and an increase in public education. Which of the following views most perpetuates the stigma? a) Mental illnesses have biochemical bases. b) Mental illness is hereditary. c) Clients can recover from mental illness if they have willpower. d) Clients cannot prevent mental illness if they want to do so.

C

A client has experienced an ischemic stroke that has damaged the temporal (lateral and superior portions) lobe. Which of the following deficits would the nurse expect during assessment of this client? a) Lack of deep tendon reflexes b) Limited attention span and forgetfulness c) Auditory agnosia d) Hemiplegia or hemiparesis

C

A client has recently brought her elderly mother home to live with her family. The client states that her mother has moderate Alzheimer's disease and asks about appropriate activities for her mother. The nurse tells the client to a) Turn off lights at night so that the mother differentiates night and day. b) Encourage the mother to take responsibility for cooking and cleaning the house. c) Ensure that the mother does not have access to car keys or drive an automobile. d) Allow the mother to smoke cigarettes outside on the porch without supervision.

C

A client suspects that he will not live. However, others talk about only pleasant matters with him and maintain a persistently cheerful facade around him. The nurse anticipates that the client will most likely feel which of the following as a result of such behavior? a) Independence. b) Relief. c) Isolation. d) Hopefulness.

C

A client who recently experienced a stroke tells the nurse that he has double vision. Which nursing intervention is the most appropriate? a) Instill artificial tears. b) Encourage the client to close his eyes. c) Alternatively patch one eye every 2 hours. d) Turn out the lights in the room.

C

A client with a diagnosis of pernicious anemia comes to the clinic complaining of numbness and tingling in his arms and legs. What do these symptoms indicate? a) Loss of vibratory and position senses b) Severity of the disease c) Neurologic involvement d) Insufficient intake of dietary nutrients

C

A client with ascites and peripheral edema is at risk for impaired skin integrity. To prevent skin breakdown, the nurse should: a) Elevate the lower extremities. b) Massage the abdomen once a shift. c) Use an alternating air pressure mattress. d) Institute range-of-motion (ROM) exercise every 4 hours.

C

A client with diabetic ketoacidosis has been brought into the ED where you practice nursing. Which of the following interventions is not a goal in the initial medical treatment of diabetic ketoacidosis? a) Administer isotonic fluid at a high volume. b) Administer potassium replacements. c) Administer glucose. d) Monitor serum electrolytes and blood glucose levels.

C

A middle-age woman is mentally preparing for the death of her mother. This is termed a) Bereavement b) Grieving c) Anticipatory grieving d) Loss

C

A nurse is assessing a client for neurologic impairment after a total hip replacement. Which of the following would indicate impairment in the affected extremity? a) Coolness to the touch b) Diminished capillary refill c) Inability to move d) Decreased distal pulse

C

A nurse is caring for a client with laceration wounds on the knee. The nurse notes that the client is in remodeling phase of wound repair. Which of the following statements describes this phase of wound recovery? a) Period during which new cells fill and seal a wound b) Physiological defense immediately after the tissue injury c) Period during which the wound undergoes changes and maturation d) Process by which damaged cells recover and re-establish normal function

C

A nurse is caring for a patient hospitalized with an exacerbation of chronic gastritis. What health promotion topic should the nurse emphasize? a) Techniques for positioning correctly to promote gastric healing b) Strategies for maintaining an alkaline gastric environment c) Strategies for avoiding irritating foods and beverages d) Safe technique for self-suctioning

C

A nurse is caring for a patient with cirrhosis secondary to heavy alcohol use. The nurse's most recent assessment reveals subtle changes in the patient's cognition and behavior. What is the nurse's most appropriate response? a) Ensure that the patient's sodium intake does not exceed recommended levels. b) Inform the primary care provider that the patient should be assessed for alcoholic hepatitis. c) Report this finding to the primary care provider due to the possibility of hepatic encephalopathy. d) Implement interventions aimed at ensuring a calm and therapeutic care environment.

C

A nurse is cleaning the wound of a gunshot victim. Which of the following is a recommended guideline for this procedure? a) Once the wound is cleaned, dry the area with an absorbent cloth. b) Use clean technique to clean the wound. c) Clean the wound from the top to the bottom and center to outside. d) Clean the wound from the bottom to the top and outside to center.

C

A nurse is completing a neurological assessment and determines that the client has significant visual deficits. A brain tumor is considered. Considering the functions of the lobes of the brain, which area will most likely contain the neurologic deficit? a) Parietal b) Frontal c) Occipital d) Temporal

C

A nurse is conducting a physical examination of a 4-month-old child. Upon assessing the pupils, the nurse notes a white pupil opening in the left eye. The parents express concern that the infant is unable to grasp objects placed nearby and does not respond to facial expressions. The parents ask the nurse what is wrong with their child's vision. What is the best response by the nurse? a) "Unfortunately, your child is going blind in the left eye." b) "There's a cataract in the left eye that should be surgically removed." c) "There appears to be something obstructing your child's vision." d) "Your child has an eye infection that can easily be treated."

C

A nurse is ordered to apply a transcutaneous electrical nerve stimulation (TENS) unit to a patient recovering from abdominal surgery. Which of the following is a consideration when using this device? a) A TENS unit is applied intermittently throughout the day and should not be worn for extended periods of time. b) TENS is most beneficial when used to treat pain that is generalized. c) TENS involves the electrical stimulation of large-diameter fibers to inhibit the transmission of painful impulses carried over small-diameter fibers. d) TENS is an invasive technique for providing pain relief

C

A nurse is preparing to receive a client in post-anesthesia care unit (PACU). The client is diabetic and has undergone knee surgery. Which information would be most important for the receiving nurse to obtain to develop an appropriate plan of care for this client? a) Information about allergic agents b) Chronic disease history c) Amount of blood loss d) Environment of the operating room

C

A nurse is teaching a patient about the types of chronic liver disease. The patient's teaching is determined to be effective based on the correct identification of which of the following types of cirrhosis caused by scar tissue surrounding the portal areas? a) Biliary cirrhosis b) Postnecrotic cirrhosis c) Alcoholic cirrhosis d) Compensated cirrhosis

C

A patient who is obese is exploring bariatric surgery options and presented to a bariatric clinic for preliminary investigation. The nurse interviews the patient, analyzing and documenting the data. Which of the following nursing diagnoses may be a contraindication for bariatric surgery? a) Disturbed Body Image Related to Obesity b) Anxiety Related to Surgery c) Deficient Knowledge Related to Risks and Expectations of Surgery d) Chronic Low Self-Esteem Related to Obesity

C

A patient with type 1 diabetes mellitus is being taught about self-injection of insulin. Which of the following facts about site rotation should the nurse include in the teaching? a) Avoid the abdomen because absorption there is irregular. b) Rotate sites from area to area every other day. c) Use all available injection sites within one area. d) Choose a different site at random for each injection.

C

An elderly client with Parkinson's disease is unable to take care of himself. The client frequently soils his bed and is unable to clean himself independently. How should the nurse in this case ensure the client's perineal care? a) Cleanse using a cotton cloth and warm water b) Provide the client with a bed pan or a jar to collect the urine c) Cleanse to remove secretions from less-soiled to more-soiled areas d) Use tissue rolls to clean the client's perineal area

C

An elderly patient has been admitted to the hospital with dehydration, and the nurse has inserted a peripheral intravenous line into the patient's forearm in order to facilitate rehydration. What type of dressing should the nurse apply over the patient's venous access site? a) A gauze dressing premedicated with antibiotics b) A dressing with a nonadherent coating c) A transparent film d) A gauze dressing precut halfway to fit around the IV line

C

An older adult client who is in a long-term care facility tells the nurse. "I'm not eating that, it's poisoned." The nurse interprets this as which manifestation of altered sensory perception? a) Withdrawal b) Sensory deficit c) Delusion d) Hallucination

C

How would a nurse assess a patient for pupillary accommodation? a) Ask the patient to focus on a finger and move the patient's eyes through the six cardinal positions of gaze. b) Ask the patient to read the smallest possible line of letters on the Snellen chart. c) Ask the patient to focus on an object as it is brought closer to the nose. d) Using an ophthalmoscope, check the red reflex.

C

In assessing an adolescent client at an outpatient clinic, the nurse is able to recognize that depression in adolescents is often which of the following? a) A sign that the teenager needs to be admitted to the hospital b) Situational and not as serious as that of adult clients c) Often masked by aggressive behaviors d) Similar in symptomology to that of adult clients

C

In contrast to anxiety, fear is characterized by which of the following? a) A real, rather than perceived, threat b) Short-term resolution c) A cognitive response to a known threat d) The creation of an action plan to deal with a perceived threat

C

In the 1970s, in children with cancer, how did guided imagery help? a) Thinking of cancer-free days makes the client feel better. b) Concentration controls cancer. c) Thinking of white cells destroying cancer increases cell count. d) Thinking happy thoughts makes the client forget the pain.

C

Loud, persistent noise has been found to cause all of the following symptoms except a) increased heart rate. b) increased blood pressure. c) decreased GI activity. d) constriction of peripheral blood vessels.

C

Nursing students are reviewing different types of mental health problems in the older adult population. They demonstrate understanding of this information when they identify which condition as the most common affective disorder? a) Anxiety b) Phobias c) Depression d) Schizophrenia

C

Susan Albert has fractured her femur and is being seen in the emergency department. As you are assessing the area, you notice there is a grating sound that you suspect is bone ends moving over one another. What would this be called? a) Deformity b) Spasm c) Crepitus d) False motion

C

The client identifies three strategies for minimizing leakage of an ileostomy bag. What type of outcome is this an example of? a) Physiologic outcome b) Psychomotor outcome c) Cognitive outcome d) Affective outcome

C

The client is waiting in a triage area to learn the medical status of his family following a motor vehicle accident. The client is pacing, taking deep breaths, and wringing the hands. Considering the effects in the body systems, what effects does the nurse anticipate in the liver? a) The liver will maintain a basal rate of functioning. b) The liver will produce a toxic by product in relation to stress. c) The liver will convert glycogen to glucose for immediate use. d) The liver will cease function and shunt blood to the heart and lungs.

C

The nurse is assessing a neglected child brought to the emergency department. The grandmother of the child reports that the child remains in the crib and is removed from the crib only when the child is fed. During the time in the crib, what is the child most likely to have experienced? a) Kinesthesia b) Stereognosis c) Sensory deprivation d) Adaptation

C

The nurse is caring for a client is scheduled for chemotherapy followed by auto logous stem cell transplant. Which of the following statements by the client indicates a need for further teaching? a) "I will need to be in protective isolation for up to 3 months after treatment." b) "I will receive chemotherapy until most of the cancer is gone, and then I will get my own stem cells back." c) "I hope they find a bone marrow donor who matches." d) "The doctor will remove cells from my bone marrow before beginning chemotherapy."

C

The nurse is reading the previous shift's documentation of an open area on the patient's sacrum. The wound is documented as a partial-thickness wound whose etiology is pressure. The nurse anticipates the assessment of the patient's sacrum will reveal a pressure ulcer in which of the following stages? a) Stage III b) Stage I c) Stage II d) Stage IV

C

The nurse teaches a client about heat and cold treatments to manage arthritis pain. Which of the following client statements indicates that the client still has a knowledge deficit? a) "Ten to 15 minutes per application is the maximum time for cold applications." b) "I can use heat and cold as often as I want." c) "Heat-producing liniments can be used with other heat devices." d) "With heat, I should apply it for no longer than 20 minutes at a time."

C

When asked how she cut her finger, a client with Alzheimer's disease says, "While cutting flowers in our garden." The client's husband later tells the nurse that they do not have a flower garden. The nurse interprets the client's statement as which of the following? a) Flight of ideas. b) Disorientation. c) Confabulation. d) Displacement.

C

When caring for a client who is on intravenous therapy, the nurse observes that the client has developed redness, warmth, and discomfort along the vein. Which of the following interventions should the nurse perform for this complication? a) Position the client on the left side b) Apply antiseptic and a dressing c) Apply a warm compress d) Elevate the client's head

C

When completing a nursing assessment on a client admitted with a neck injury, which of the following findings would indicate an incomplete spinal cord injury (SCI)? a) Presence of paralytic ileus and urinary retention b) Immediate flaccid paralysis that persists for months c) Evidence of voluntary motor and sensory function below the level of injury d) Presence of spinal shock response with hypertension and tachycardia

C

When teaching a client with an extracapsular hip fracture scheduled for surgical internal fixation with the insertion of a pin, the nurse bases the teaching on the understanding that this surgical repair is the treatment of choice. Which of the following explains the reason? a) The risk of infection at the site is lessened. b) Hemorrhage at the fracture site is prevented. c) The client is able to be mobilized sooner. d) Neurovascular impairment risk is decreased.

C

Which of the following characteristics would put a client at the greatest risk for impaired wound healing after abdominal surgery? a) Age 60 years, with peripheral vascular disease. b) Age 75 years. c) Age 30 years, with poorly controlled diabetes. d) Age 55 years, with myocardial infarction.

C

Which of the following factors is a priority when evaluating discharge plans for an older adult after a lower left lobectomy for lung cancer? a) The client's knowledge of the causes of lung cancer. b) The client's ability to do home blood pressure monitoring. c) Support available for assisting the client at home. d) The distance the client lives from the hospital.

C

Which of the following is consistent with acute otitis media? a) It is usually caused by a fungal infection. b) It is a relatively uncommon childhood infection. c) Conductive hearing loss may occur. d) The infection usually lasts more than 6 weeks.

C

Which of the following may occur with respiratory acidosis? a) Decreased blood pressure b) Third spacing c) Increased intracranial pressure (ICP) d) Decreased pulse

C

Which of the following should the nurse include in the teaching plan for a client with seizures who is going home with a prescription for gabapentin? a) Store gabapentin in the refrigerator. b) Take gabapentin with an antacid to protect against ulcers. c) Notify the physician if vision changes occur. d) Take all the medication until it is gone.

C

Which of the following statements indicates the client has understood the instructions to follow at home after cataract surgery? a) "I should keep my protective eye shield in place at all times." b) "I can lift what I want." c) "I should not bend over to pick up objects from the floor." d) "I may not watch television for 3 weeks."

C

Which of the following terms refers to a condition characterized by destruction of the melanocytes in circumscribed areas of the skin? a) Hirsutism b) Telangiectases c) Vitiligo d) Lichenification

C

Which statement by the client indicates further teaching about epidural anesthesia is necessary? a) "I will be able to hear the surgeon during the surgery." b) "I will lose the ability to move my legs." c) "I will become unconscious." d) "A needle will deliver the anesthetic into the area around my spinal cord."

C

Which statement describes benign paroxysmal positional vertigo (BPPV)? a) BPPV is caused by tympanic membrane rupture. b) The onset of BPPV is gradual. c) The vertigo is usually accompanied by nausea and vomiting. d) BPPV is stimulated by the use of certain medication such as acetaminophen.

C

You are caring for a 68-year-old female patient who weighs 108 pounds. The patient has been worried about regularity of her bowel movements. You have just placed a regular bedpan under the patient and the patient states, "Please leave the bedpan in place until I have a bowel movement." Which of the following responses is most appropriate? a) "Leaving the bedpan in place for a long period of time is unusual. Are you worried that staff will not respond in a timely manner when you need to have a bowel movement?" b) "Remember, you can also urinate into the bedpan." c) "I will check back in 10 minutes and remove the bedpan for a period of time even if you have not had a bowel movement." d) "Turn your call light on when you want me to come back into the room."

C

You are caring for a patient who has dysphagia and is unable to eat independently. You are preparing to assist the patient in eating a meal. Which of the following actions is appropriate? a) Arrange food items in a clock face pattern and inform the patient what time on a clock corresponds to each food item. b) Create a positive social environment by asking the patient about childhood food memories. c) Speak to the patient but limit the need for the patient to respond verbally while chewing and swallowing. d) Encourage the patient to eat using a consistent, efficient pace to prevent hot foods from becoming too cool and cool foods from becoming too warm.

C

A patient present to the ED following a work-related injury to the left hand. The patient has an avulsion of the left ring finger. Which of the following correctly describes an avulsion? a) Incision of the skin with well-defined edges, usually longer than deep b) Skin tear with irregular edges and vein bridging c) Tearing away of tissue from supporting structures d) Denuded skin

C An avulsion is described as a tearing away of tissue from supporting structures. A laceration is a skin tear with irregular edges and vein bridging. Abrasion is denuded skin. A cut is an incision of the skin with well-defined edges, usually longer than deep.

A postoperative client describes the following during a transfer, "I feel like something just popped." The nurse immediately assesses for a) Herniation b) Infection c) Dehiscence d) Evisceration

C Dehiscence is a total or partial disruption in wound edges. Clients often report feeling the incision has given way.

Bill Jenkins has suffered from a burn on his leg related to an engine fire. Burn depth is determined by assessing the color, characteristics of the skin, and sensation in the area. When the burn area was assessed, it was determined that he felt no pain in the area and that it appeared charred. What depth of burn injury would he be said to have? a) Superficial partial-thickness and deep partial-thickness (second degree) b) Fourth degree c) Full thickness (third degree) d) Superficial (first degree)

C Full-thickness (third degree) burn destroys all layers of the skin and consequently is painless. The tissue appears charred or lifeless. Superficial (first degree) burn is similar to a sunburn. The epidermis is injured, but the dermis is unaffected. Superficial partial-thickness burn heals within 14 days, with possibly some pigmentary changes but no scarring. The deep partial-thickness (second degree) burn takes more than 3 weeks to heal, may need debridement, and is subject to hypertrophic scarring. A fourth-degree burn can involve ligaments, tendons, muscles, nerves, and bone.

A hospital patient has been awakened at night by the alarm on his roommate's intravenous pump. This patient was aroused by brain action in his: a) limbic system. b) cerebellum. c) reticular activating system (RAS). d) prefrontal cortex.

C The RAS is the network that mediates arousal.

Which statement regarding heart sounds is correct? a) S1 and S2 sound fainter at the apex than at the base. b) S1 and S2 sound fainter at the base than at the apex. c) S1 is loudest at the apex, and S2 is loudest at the base. d) S1 and S2 sound equally loud over the entire cardiac area.

C The S1 sound — the "lub" sound — is loudest at the apex of the heart. It sounds longer, lower, and louder there than the S2. The S2 — the "dub" sound — is loudest at the base. It sounds shorter, sharper, higher, and louder there than the S1.

A 76-year-old client has a significant history of congestive heart failure. During his semiannual cardiology examination, for what should you, as his nurse, specifically assess? Select all that apply. a) Examine the client's joints for crepitus. b) Examine the client's eyes for excess tears. c) Monitor the client for signs of lethargy or confusion. d) Examine the client's neck for distended veins.

C, D

A client is being admitted with a spinal cord transection at C7. Which of the following assessments take priority upon the client's arrival? Select all that apply. a) Bladder function b) Reflexes c) Temperature d) Blood pressure e) Respirations

C, D, E

A client is admitted to the psychiatric unit with a diagnosis of functional neurologic symptom disorder. Since witnessing a beating at gunpoint, the client is paralyzed. Which action should the nurse initially focus on when planning this client's care? a) Helping the client identify any stressors or psychological conflicts b) Exploring personal relationships that may be related to the paralysis c) Teaching the client to deal with any limitations of the paralysis d) Helping the client identify and verbalize his/her feelings about the incident

D

A client is brought to the emergency department after injuring his right arm in a bicycle accident. The orthopedic surgeon tells the nurse that the client has a greenstick fracture of the arm. What does this mean? a) The fracture results from an underlying bone disorder. b) The fracture line extends through the entire bone substance. c) Bone fragments are separated at the fracture line. d) One side of the bone is broken and the other side is bent.

D

A client on prolonged bed rest has developed a pressure ulcer. The wound shows no signs of healing even though the client has received skin care and has been turned every 2 hours. Which factor is most likely responsible for the failure to heal? a) Low calcium level b) Inadequate massaging of the affected area c) Inadequate vitamin D intake d) Inadequate protein intake

D

A client reports to the nurse that her mother had macular degeneration and is concerned that she, too, may be at risk. The nurse tells the client to a) Reduce the amount of cigarettes smoked daily from 20 to 10. b) Stop worrying, because this condition is now curable. c) Stop worrying, because she is not at increased risk. d) Wear sunglasses with ultraviolet (UV) protection when outside.

D

A client who has had a bowel resection comes to the health center 7 days postoperatively for removal of the staples. As the nurse is cleansing the incision, the client reports of mid-incision pain. After removing three staples, the nurse observes that the incision is separating. What is the nurse's priority action? a) Remove the remaining staples, apply butterfly tapes, and document the findings. b) Apply butterfly tapes to the separated area and redress the wound immediately. c) Apply warm compresses to the painful area before removing the remaining staples. d) Stop the staple removal, cover the incision, and report the findings to the physician.

D

A client with dementia who prefers to stay in his room has been brought to the dayroom. After 10 minutes, the client becomes agitated and retreats to his room again. The nurse decides to assess the conditions in the dayroom. Which is the most likely occurrence that is disturbing to this client? a) There are three staff members and one primary health care provider in the nurse's station working on charting. b) A housekeeping staff member is washing off the countertops in the kitchen, which is on the far side of the dayroom. c) There is only one other client in the dayroom; the rest are in a group session in another room. d) A relaxation tape is playing in one corner of the room, and a television airing a special on crime is playing in the opposite corner.

D

A client with quadriplegia is in spinal shock. What finding should the nurse expect? a) Hyperreflexia along with spastic extremities b) Spasticity of all four extremities c) Positive Babinski's reflex along with spastic extremities d) Absence of reflexes along with flaccid extremities

D

A client with schizophrenia is admitted to the psychiatric unit of a local hospital. During the next several days, the client is seen laughing, yelling, and talking to himself. This behavior is characteristic of: a) delusion. b) looseness of association. c) illusion. d) hallucination.

D

A nurse is brushing the hair of a client admitted to the healthcare facility following a fracture in the hand. The nurse implements this action based on the understanding that brushing the hair achieves which of the following? a) Cleans hair and scalp b) Removes excess oil c) Cleans the hair of dirt d) Facilitates oil distribution

D

A nurse is caring for a client during barbiturate therapy. The client receiving this drug should be evaluated for which of the following? a) Prolonged bleeding time b) Hepatotoxicity c) Poor drug absorption d) Physical dependence

D

A nurse is removing sutures from the surgical wound of a patient after an appendectomy and notices that the sutures are encrusted with blood and difficult to pull out. What would be the appropriate intervention in this situation? a) Wash the sutures with warm, sterile water and an antimicrobial soap before removing them. b) Pick the crusts off the sutures with the forceps before removing them. c) Do not attempt to remove the sutures because they need more time to heal. d) Moisten sterile gauze with sterile saline to loosen crusts before removing sutures.

D

A nurse is teaching a client who has facial muscle weakness and has recently been diagnosed with myasthenia gravis. The nurse should teach the client that myasthenia gravis is caused by: a) upper and lower motor neuron lesions. b) decreased conduction of impulses in an upper motor neuron lesion. c) genetic dysfunction. d) a lower motor neuron lesion.

D

A nurse notes a number of laceration wounds around the cervix of the uterus due to childbirth. How could the nurse describe the laceration wound in the client's medical record? a) A clean separation of skin and tissue with a smooth, even edge b) A wound in which the surface layers of skin are scraped away c) A shallow crater in which skin or mucous membrane is missing d) A separation of skin and tissue in which the edges are torn and irregular

D

A parent of a child with a moderate head injury asks the nurse, "How will you know if my child is getting worse?" The nurse should tell the parents that best indicator of the child's brain function is: a) Reactions of the pupils. b) Motor strength. c) The vital signs. d) Level of consciousness.

D

A patient returning from the operating room is unconscious. What guidelines should the nurse consider when communicating with this patient? a) Gently shake the patient's hand or arm before speaking to him or her. b) Provide loud environmental stimuli to assist in arousing the patient. c) There are no guidelines to consider because the patient cannot hear the nurse. d) Talk to the patient in a normal tone of voice.

D

A patient with uncontrolled hypertension experienced a stroke 1 week ago, leading to significant motor losses. A successful and normal adaptive response to these new limitations is evident if the patient: a) repeatedly states, "It is what it is." b) refuses to participate in physiotherapy. c) changes the subject when the nurse addresses activities of daily living (ADLs). d) exhibits signs of grief.

D

A postmastectomy patient is due to come up to the postsurgical care unit from postanesthetic recovery. Which of the following nursing actions should the nurse prioritize when attempting to establish an effective relationship with her? a) Address the patient's potential learning needs. b) Assess the patient's knowledge of her activity limitations. c) Explain and answer questions about the Health Insurance Portability and Accountability Act (HIPAA). d) Recognize and address the patient's anxiety.

D

A teenager asks advice from a nurse about getting a tattoo. When the nurse is providing education, which statement about tattoos is a common misconception? a) Human immunodeficiency syndrome (HIV) is a possible risk factor. b) Hepatitis B is a possible risk factor. c) Allergic response to pigments is a possible risk factor. d) Tattoos are easily removed with laser surgery.

D

An 86-year-old patient with a diagnosis of late-stage Alzheimer disease requires full assistance with transfers to and from his bed. Which of the following nursing actions is most likely to promote safe handling of this patient? a) Ask for the patient's input on the timing and technique for transfers. b) Ask for the patient's feedback frequently during transfers. c) Post written instructions at the patient's bedside to supplement spoken instructions. d) Provide brief, clear instructions to the patient that are phrased positively.

D

An elderly client is complaining of dry, itching skin. The nurse should assess a) When the client's last tub bath was b) When the severe itching occurs c) What linens they are using d) How often the client is bathing

D

Dementia is a disorder that progresses over several years, with increasing confusion, forgetting family, and disorientation in familiar surroundings. A common problem with dementia patients is sundowning syndrome, which is described as: a) increasing sleeplessness at night because the patient cat-naps during the day. b) occasional onset of marked confusion, wandering and feeling lost during the afternoon, before sunset. c) a behavior change at sunset as the patient becomes more fatigued, listless, and disoriented. d) habitual agitation, restlessness, and confusion that occurs after dark.

D

For healing by secondary intention, a client's wound has been packed with medicated dressings. The nurse assesses the wound. Which finding indicates wound healing? a) The tissue surrounding the wound is red and hot. b) The skin around the wound is edematous. c) The wound drainage is serous. d) The granulation tissue is at the wound edges.

D

Hypothermia may occur as a result of a) being young. b) the infusion of warm fluids. c) increased muscle activity. d) open body wounds.

D

In group therapy, a client who has used I.V. heroin every day for the past 14 years says, "I don't have a drug problem. I can quit whenever I want. I've done it before." Which defense mechanism is the client using? a) Identification b) Compensation c) Rationalization d) Denial

D

Mr. Jacobs has been recently diagnosed with diabetes mellitus after receiving emergency treatment for a hyperglycemic episode. Which of Mr. Jacobs' actions indicates that he has achieved a cognitive outcome in the management of his new health problem? a) Mr. Jacobs can demonstrate the correct technique for using his new glucometer b) Mr. Jacobs' blood sugars have been maintained within acceptable range in the days prior to discharge c) Mr. Jacobs expresses a desire to change the way that he eats and the amount of exercise he performs d) Mr. Jacobs is able to explain when and why he needs to check his blood sugar

D

Mr. Singh's wife has recently been diagnosed with early-stage Alzheimer's disease and he has asked the nurse to recommend Web sites that may supplement his learning about her diagnosis. How should the nurse respond to Mr. Singh's request? a) Encourage Mr. Singh to avoid online resources due to the unregulated nature of the Internet. b) Provide Mr. Singh with print-based materials that are clearly referenced and reflect his learning style. c) Direct Mr. Singh to online databases such as the Cumulative Index to Nursing and Allied Health Literature (CINAHL) d) Identify and recommend some credible Web sites appropriate to his learning needs.

D

Mrs. R. has developed an abscess following abdominal surgery and her food intake has been decreasing over the past 2 weeks. Which of the following laboratory findings may suggest the need for nutritional support? a) Low random blood glucose levels b) Proteinuria c) Increased white blood cells d) Low serum albumin levels

D

On a routine visit to the physician, a client with chronic arterial occlusive disease reports that he's stopped smoking after 34 years. To relieve symptoms of intermittent claudication, a condition associated with chronic arterial occlusive disease, which additional measure should the nurse recommend? a) Engaging in anaerobic exercise b) Reducing daily fat intake to less than 45% of total calories c) Abstaining from foods that increase levels of high-density lipoproteins (HDLs) d) Taking daily walks

D

Painless chancre lesions are associated with which systemic disease? a) Urticaria b) Kaposi's sarcoma c) Psoriasis d) Syphilis

D

Prior to administering tissue plasminogen activator (t-PA), the nurse should assess the client for which of the following? a) Age greater than 60 years. b) History of heart failure. c) Cigarette smoking. d) History of cerebral hemorrhage.

D

Rebleeding may occur from a peptic ulcer and often warrants surgical interventions. Signs of bleeding include which of the following? a) Bradypnea b) Hypertension c) Bradycardia d) Mental confusion

D

The client with a diagnosis of bipolar disorder, manic phase, states to the nurse, "I'm the Queen of England. Bow before me." The nurse interprets this statement as important to document as which of the following areas of the mental status examination? a) Psychomotor behavior. b) Attitude toward the nurse. c) Mood and affect. d) Thought content.

D

The nurse identifies the nursing diagnosis of deficient knowledge related to a new hearing aid for a client. After teaching a client about caring for his new hearing aid, the nurse determines that the outcome has been achieved when the client states which of the following? a) "I should insert the ear mold when it is wet." b) "I need to wipe the ear mold daily with a moist washcloth." c) "I should wash the receiver with soap and water once a week." d) "I need to keep my ear canal clean and dry."

D

The nurse is caring for Kristin Brook, a 53-year-old patient who suffered a traumatic brain injury in a skiing accident. Kristin breathes on her own, is very drowsy, but can be aroused by extreme or repeated stimuli. The nurse documents that Kristin's level of consciousness is which of the following? a) Coma b) Asleep c) Somnolence d) Stupor

D

The nurse is caring for a client with a panic attack. Which nursing intervention is most helpful for this client? a) Encourage the client to identify what precipitated the attack b) Encourage the client to verbalize any fears, feelings, or concerns c) Encourage the client to learn relaxation techniques d) Stay with the client and remaining calm, confident, and reassuring

D

The nurse is instructing a community class when a student asks, "How does someone get super strength in an emergency?"The nurse is correct to instruct on the action of which system? a) Musculoskeletal system b) Endocrine system c) Parasympathetic nervous system d) Sympathetic nervous system

D

The nurse is teaching the parents of a 7-year-old child who has been newly diagnosed with absence seizures. The nurse is discussing behavior that may indicate seizure activity. Which of the following information is most appropriate for the nurse to teach about absence seizure activity? a) "Look for rapid contraction and relaxation of the skeletal muscles." b) "Look for the eyes to roll back, a loss of consciousness, and convulsions." c) "Look for the body to tense and a loud moan or scream to occur." d) "Look for brief episodes of twitching accompanied by disorientation."

D

The nurse is working with a patient who is in a stressful situation. The nurse evaluates the patient's resiliency by assessing the patient's ability to do which of the following? a) Verbalize feelings of anger b) Respond with strong emotions c) Admit past mistakes d) Continue to function well

D

The nurse observes the client and notes a shuffling gait. The nurse recognizes this finding is consistent with: a) Lower motor neuron disease b) Paget's disease c) Scoliosis d) Parkinson's disease

D

The nurse uses Montgomery straps *primarily* to achieve which of the following client outcomes? a) The client is free from wandering. b) The client is free from falls. c) The client is free from bruises. d) The client is free from skin breakdown.

D

The rate at which I.V. fluids are infused is based on the burn client's: a) Height and weight and BSA burned. b) Lean muscle mass and body surface area (BSA) burned. c) Total BSA and BSA burned. d) Total body weight and BSA burned.

D

To assess the effectiveness of cardiac compressions during adult cardiopulmonary resuscitation (CPR), a nurse should palpate which pulse site? a) Apical b) Brachial c) Radial d) Carotid

D

When caring for a patient with cirrhosis, which of the following symptoms should the nurse report immediately? a) Fatigue and weight loss b) Anorexia and dyspepsia c) Diarrhea or constipation d) Change in mental status

D

When examining a client who has abdominal pain, a nurse should assess: a) the symptomatic quadrant first. b) any quadrant first. c) the symptomatic quadrant either second or third. d) the symptomatic quadrant last.

D

Which intervention should the nurse implement, when caring for the client who complains of phantom limb pain two months after amputation? a) Assess the stump for signs and symptoms of bleeding. b) Reposition the stump, elevating it on two pillows. c) Assess the stump for signs and symptoms of infection. d) Reassure the client that phantom pain is common.

D

Which of the following is a priority to include in the plan of care for a client with Alzheimer's disease who is experiencing difficulty processing and completing complex tasks? a) Demonstrating for the client how to do the task. b) Maintaining routine and structure for the client. c) Repeating the directions until the client follows them. d) Asking the client to do one step of the task at a time.

D

Which of the following is a realistic short-term goal to be accomplished in 2 to 3 days for a client with delirium? a) Establish normal bowel and bladder function. b) Resume a normal sleep-wake cycle. c) Explain the experience of having delirium. d) Regain orientation to time and place.

D

Which of the following is most likely to cause the client to experience postoperative nausea and vomiting? a) Mastectomy of the left breast. b) Total hip replacement. c) Mitral valve repair. d) Abdominal hysterectomy.

D

Which of the following nursing intervention can prevent a client from experiencing autonomic dysreflexia? a) Assessing laboratory test results as ordered b) Administering zolpidem tartrate c) Placing the client in the Trendelenburg's position d) Monitoring the patency of an indwelling urinary catheter

D

Which of the following pieces of equipment is used to perform Weber's test and Rinne's test? a) Cotton-tipped applicator b) Laryngeal mirror c) Reflex hammer d) Tuning fork

D

Which of the following terms refers to a protrusion of abdominal organs through the surgical incision? a) Hernia b) Dehiscence c) Erythema d) Evisceration

D

Which term describes a surgical procedure to release constricting muscle fascia so as to relieve muscle tissue pressure? a) Arthroplasty b) Arthrodesis c) Osteotomy d) Fasciotomy

D

While interviewing a client during a routine health visit, the client informs the nurse that he meditates regularly to combat stress. The nurse demonstrates understanding of this technique as involving which of the following? a) Balancing Yin and Yang b) Consciously directing energy exchange c) Stimulating the meridians d) Engaging in deep personal thought and reflection

D

You are caring for a patient who is on bed rest and was just turned to the left side. Which of the following actions should you take next to decrease the risk of impaired skin integrity? a) Cover the patient with the bed linens. b) Place the call bell within reach. c) Assess for pain. d) Pull the shoulder blade forward and out from under the patient.

D

You are completing the admission of a confused older adult to the Alzheimer's unit. As you remove her arm from the gown, you notice bruises in various stages on both sides of her shoulders and arms. As you continue with your examination, the daughter explains loudly: "Mom is always banging into the door." You acknowledge her, complete the assessment, and chart your findings. What else should you do? a) Go back and talk to the patient by herself. b) Report you suspicion of elder abuse to the Abuse Hotline. c) Mark the front of the paper record or flag the electronic record with an abuse alert. d) Discuss your concerns with your immediate supervisor.

D

When the bladder contains 300 mL or more of urine, this is referred to as a) renal clearance b) anuria c) specific gravity d) functional capacity

D A marked sense of fullness and discomfort with a strong desire to void usually occurs when the bladder contains 350 mL or more of urine, referred to as the "functional capacity." Anuria is a total urine output of less than 50 mL in 24 hours. Specific gravity reflects the weight of particles dissolved in the urine. Renal clearance refers to the ability of the kidneys to clear solutes from the plasma.

A nurse is assessing a client who is receiving clozapine. The nurse reviews the chart. What should the nurse do next? Vital Signs-RR: 24 P: 148 T 98.3 F B/P 120/84 a) Give the clozapine, and tell the client to lie down. b) Withhold the clozapine, and tell the client to go to an exercise group. c) Administer the clozapine, and notify the physician. d) Withhold the clozapine, and notify the primary care provider.

D Because clozapine can cause tachycardia, the nurse should withhold the medication if the pulse rate is greater than 140 bpm and notify the physician. Giving the drug or telling the client to exercise could be detrimental to the client.

Upon assessment of a patient with multiple sclerosis, the nurse notes that the patient is difficult to understand due to an inability to produce speech sounds related to poor respiratory control and impaired movement of the lips and tongue. The nurse will document this finding on the patient's chart as: a) impaired hearing. b) aphasia. c) esophageal speech. d) dysarthria.

D Dysarthria interferes with normal control of the speech mechanism. Speech may be slurred or difficult to understand due to lack of ability to produce speech sounds correctly, maintain good breath control, or coordinate the movements of the lips, tongue, palate, and larynx. Aphasia results in a reduced ability to understand what others are saying, to express oneself, or to be understood. Esophageal speech is often learned after a laryngectomy and requires the use of an electronic device or surgical implant of a voice prosthesis. This scenario does not define a hearing problem.

Which of the following is a vesicant? a) Sarin b) Hydrogen cyanide c) Chlorine d) Nitrogen mustard

D Examples of vesicants are phosgene, nitrogen mustard, and sulfur mustard. Sarin is a nerve agent. Hydrogen cyanide is a blood agent. Chlorine is a pulmonary agent.

A longitudinal tear or ulceration in the lining of the anal canal is termed a (an) a) anal fistula. b) anorectal abscess. c) hemorrhoid. d) anal fissure.

D Fissures are usually caused by the trauma of passing a large, firm stool or from persistent tightening of the anal canal secondary to stress or anxiety (leading to constipation).

A nurse is reviewing a client's laboratory test results. Which electrolyte is the major cation controlling a client's extracellular fluid (ECF) osmolality? a) Potassium b) Chloride c) Calcium d) Sodium

D Sodium, the major ECF cation, maintains ECF osmolality. Potassium is the major cation in intracellular fluid. Chloride is the major anion in the ECF. Calcium, found primarily in the intravascular fluid compartment of ECF, is the major cation involved in the structure and function of the teeth and bones.


Kaugnay na mga set ng pag-aaral

Vistas Volume 2: Chapter 8 - ¿Qué comida es?, Categorías, & ¿Cuándo lo comes?

View Set

Assessment and Management of Patients With Hepatic Disorders

View Set

VHL Chapters/Unidades 1-14 Spanish 1-3 Vocab

View Set

Unit 4: Chapter 12 (Nervous System III)

View Set

Intro to Business Chapter 8 Vocab.

View Set

Chapter 16 - Financial Leverage and Capital Policy

View Set

Reflexive Verbs: Spanish Week 10 Review Part & Week 11 Review (Module 3)

View Set